Đến nội dung

tuan101293 nội dung

Có 316 mục bởi tuan101293 (Tìm giới hạn từ 29-04-2020)



Sắp theo                Sắp xếp  

#221024 Mệnh đề tương đương

Đã gửi bởi tuan101293 on 20-11-2009 - 21:26 trong Đại số

Cho hàm số y=f(x)= ax+b
Biết f(-1)<f(-2) ; f(1)>f(2) và f(1999)=2000 . Tính : f(2010)

Bài này ko làm đc đâu em ơi.
hai cái đầu suy ra a<0.
với mỗi a<0 ta luôn chọn đc 1 số b:f(1999)=2000
khi đó f(2010) thay đổi.



#221025 Mệnh đề tương đương

Đã gửi bởi tuan101293 on 20-11-2009 - 21:39 trong Đại số

+,nếu tồn tại i khác j mà f(j)=f(i) suy ra i=-f(f(m+1))-f(m+f(f(i)))=-f(f(m+1))-f(m+f(f(j)))=j suy ra vô lý
suy ra f đơn ánh
+,cho m=0 suy ra f(f(f(n)))=-f(f(1))-n
suy ra f là toàn ánh
suy ra f là song ánh



#251860 Sắp đến Tết rùi

Đã gửi bởi tuan101293 on 21-01-2011 - 21:20 trong Góc giao lưu

Ước đỗ đại học ^_^



#251865 Bất đẳng thức và cực trị (THCS)

Đã gửi bởi tuan101293 on 21-01-2011 - 21:44 trong Bất đẳng thức và cực trị

bài nữa cũng khó!
Cho a,b,c là 3 cạnh của 1 tam giác S là diện tích tam giác
CM:
$\dfrac{ab\sqrt{ab}}{a+b}+\dfrac{bc\sqrt{bc}}{b+c}+\dfrac{ca\sqrt{ca}}{c+a}\geq 2\sqrt{3}S$

Xài bdt Finsler hadwiger ta có
$4\sqrt{3}S\le 2\sum ab-\sum a^2$
ta sẽ CM
$2\sum ab-\sum a^2\le \sum \dfrac{2ab\sqrt{ab}}{a+b}$
tương đương (biến đổi S.O.S)
$\sum (\sqrt{a}-\sqrt{b})^2*[\dfrac{(a+b)(\sqrt{a}+\sqrt{b})^2-2ab}{2(a+b)}]\ge 0$
(đúng)
ĐPCM
còn bài Trung quốc phía sau thì chỉ cần fang công thức cos theo 3 cạnh,svac+schur là ra ngay



#251909 Bất đẳng thức và cực trị (THCS)

Đã gửi bởi tuan101293 on 22-01-2011 - 19:51 trong Bất đẳng thức và cực trị

Có ai giúp em với !
"bdt Finsler hadwiger " là gì thế??
em chưa học cái này

Đây nè em
http://planetmath.or...Inequality.html



#253365 Bất đẳng thức và cực trị (THCS)

Đã gửi bởi tuan101293 on 17-02-2011 - 15:26 trong Bất đẳng thức và cực trị

bài này ko khó nè!
$Let a,b,c > 0 such that : a+b+c = 1 . Prove that :$

$\sum \dfrac{a^2}{b} \geq 3\sum a^2$

Ta có
$VT=\sum_{cyc} \dfrac{a^4}{a^2b}\ge \dfrac{(\sum a^2)^2}{\sum_{cyc} a^2b} $
tức là ta sẽ CM
$\sum a^2\ge 3\sum_{cyc} a^2b$
mà $\sum a^2=(\sum a^2)(\sum a)$
nên ta cần cm
$\sum a^3+\sum_{cyc} ab^2\ge 2\sum_{cyc} a^2b$
đúng vì $a^3+ab^2\ge 2a^2b$ ,....
ĐPCM
(ký hiệu $\sum_{cyc} a^2b=a^2b+b^2c+c^2a$)



#253110 Bất đẳng thức và cực trị (THCS)

Đã gửi bởi tuan101293 on 11-02-2011 - 21:07 trong Bất đẳng thức và cực trị

Trong đề thi HSG Trung Quốc
Cho tam giác ABC nhọn, chứng minh
$\dfrac{cos^2 A}{cos A +1}$ + $\dfrac{cos^2 B}{cos B +1}$ + $\dfrac{cos^2 C}{cos C +1}$ $\geq$ $\dfrac{1}{2}$

http://www.artofprob...v...=59649&ml=1



#251914 Bất đẳng thức và cực trị (THCS)

Đã gửi bởi tuan101293 on 22-01-2011 - 19:59 trong Bất đẳng thức và cực trị

giúp mình!
Cho a, b, c > 0 thỏa mãn a+b+c = 3.Chứng minh rằng:
$12(\dfrac{1}{a} + \dfrac{1}{b} + \dfrac{1}{c}) \geq 4(a^3 + b^3 + c^3) + 21$
(hơi khó đấy! )

Dấu = ở đâu nhỉ??
sai đề chăng?



#242311 gjup mh bai nay vs

Đã gửi bởi tuan101293 on 29-09-2010 - 16:54 trong Phương trình, hệ phương trình và bất phương trình

Cho: a+b+c=1
a^2+b^2+c^2=1
a^3+b^3+c^3=1
tính giá trị của P=a^2002+b^2003+c^2004
Các bạn giúp mh vs nha! bài tập ôn luyên HSG đấy! nhưng đối vs các bạn thì chăks là k khó đâu

suy ra $0=(a+b+c)^2-a^2-b^2-c^2=2(ab+bc+ca)$
Bạn chú ý đẳng thức
$1-3abc=a^3+b^3+c^3-3abc=(a^2+b^2+c^2-ab-bc-ca)(a+b+c)=1$
suy ra 3abc=0
Giả sử a=0
suy ra $b+c=1$, $b^2+c^2=1$
suy ra $0=(b+c)^2-b^2-c^2=2bc$
nên b=0,c=1 (chú ý hoán vị)
suy ra P=1



#268286 Cùng nhau thảo luận về IMO 2011

Đã gửi bởi tuan101293 on 12-07-2011 - 21:21 trong Thi HSG Quốc gia và Quốc tế

Nghe từ cục cưng mà cứ thấy làm sao ý
ko bik ý tốt hay là ý j đây



#275278 Number theory Marathon

Đã gửi bởi tuan101293 on 05-09-2011 - 12:01 trong Số học

anh hiểu nhầm ý em rùi vì y thuộc Z nên $ \sqrt{1-4k^4y^2}$ là ước của 1 (ước của 1 gồm 1 và -1(loại)) từ đó em mới giải đây là cách giải của em nên em cũng sợ sai lắm (bài này trích ở đâu zậy mấy anh để em xem đáp án)

ô hay,
$\sqrt{1-4k^4y^2}$ có nguyên đâu, chỉ hữu tỉ thôi nên nó fai có dạng 1/n



#273372 Number theory Marathon

Đã gửi bởi tuan101293 on 21-08-2011 - 11:06 trong Số học

cho em giải bằng cách này được không ? cho em ý kiến nhé!!
vì x,y là nghiệm nguyên nên ta có: x=ky ( y thuộc Z , k thuộc R)
thay vào PT ta có:
$ 2k^4y^4+1=y^2 \Leftrightarrow y^2(1-2k^4y^2)=1 \Leftrightarrow y= \dfrac{1}{ \sqrt{1-2k^4y^2} } $
vì y thuộc Z nên $ \dfrac{1}{ \sqrt{1-2k^4y^2} } $ cũng thuộc Z
từ đó ta sẽ có $ \sqrt{1-2k^4y^2} =1 \Leftrightarrow k=0 hay y=0 $

chú giải sai chỗ này : vì y thuộc Z nên cái căn kia có dạng 1/n chứ ko fai là 1
***
bài này thì xài xuống thang thôi
China TST 1993
http://www.artofprob...dc926aa#p269097



#237825 Very difficult inequality

Đã gửi bởi tuan101293 on 21-08-2010 - 22:30 trong Bất đẳng thức - Cực trị

Bạn giải ra luôn đi nhé. Thấy anh Hùng nói như trên thì mình thấy rằng bạn nên giải ra luôn cho mọi người tham khảo.

Bạn có thể tham khảo file này

File gửi kèm




#238686 Tuyển tập chuyện cười toán học

Đã gửi bởi tuan101293 on 30-08-2010 - 12:13 trong Toán học lý thú

Mặc dù tên này có thể lấy kính vì tiền,nhưng nghe cũng hay đó.thks



#251952 Sequence & Equation

Đã gửi bởi tuan101293 on 23-01-2011 - 07:29 trong Dãy số - Giới hạn

1 vài bài dãy nữa :
1/Cho $f\left( n \right) = \left( {n^2 + n + 1} \right)^2 + 1 $
$u_n = \dfrac{{f\left( 1 \right).f\left( 3 \right).....f\left( {2n - 1} \right)}}{{f\left( 2 \right).f\left( 4 \right).....f\left( {2n} \right)}},n \in N^* $.Tính $\lim n\sqrt{u_n}$
********************************
Bài này thì cũ rồi
ý tưởng chỉ là phân tích f(n)
$f(n)=(n^2+1)(n^2+2n+1=(n^2+1)((n+1)^2+1)$
Giản ước đi còn
$A=\dfrac{n^2+1}{(2n+1)^2+1}$
hình như là ra cái lim kia=$\ infty $ thì fair



#253224 Sequence & Equation

Đã gửi bởi tuan101293 on 14-02-2011 - 16:25 trong Dãy số - Giới hạn

1/Tính $\lim {\dfrac{1}{\sqrt{n}}. \sum\limits_{i=1}^{n}\dfrac{1}{\sqrt{i}}}$ khi $n -> + \infty $

2/Cho dãy $\{a_n\}$ có $\lim {a_n}=a=const$.Tính $\lim {\dfrac{na_1+(n-1)a_2+...+a_n}{n^2}}$

Bài 1 thì dùng stolz:
$x_n= \sum\limits_{i=1}^{n}\dfrac{1}{\sqrt{i}}}$
$y_n=\sqrt{n}$
khi đó áp dụng stolz ta có
$\dfrac{lim (x_n)}{lim(y_n)}=\dfrac{\lim (x_n-x_{n-1})}{lim (y_n-y_{n-1})}=\dfrac{\dfrac{1}{\sqrt{n}}}{\sqrt{n}+\sqrt{n-1}}=2$
suy ra $lim x_n=+infty$ thì phải???
Bài 2 thì fang trực tiếp tiêu chuẩn teoplitz vào là ra ngay a/2



#237916 Chứng minh bđt sau

Đã gửi bởi tuan101293 on 22-08-2010 - 21:50 trong Bất đẳng thức - Cực trị

Bài 1. Cho a,b,c là các số thực dương và abc=1.
Chứng minh :
$\dfrac{a}{a+{b}^{2}+{c}^{3}}+\dfrac{b}{b+{c}^{2}+{a}^{3}}+\dfrac{c}{c+{a}^{2}+{b}^{3}} \leq 1$

Mình định dùng bunhia nhưng ko được.BDT khá đẹp nên mình hy vọng có chủ topic,hay ai đó sẽ post lời giải lên.
thks



#239013 Chứng minh bđt sau

Đã gửi bởi tuan101293 on 01-09-2010 - 21:26 trong Bất đẳng thức - Cực trị

2 BDT này sẽ đúng (có thể sai :))
$\sum a^3b^5 \ge abc \sum ab^4$
$\sum a^5b^2 \ge abc \sum a^3b$

bđt thứ 2 thì đúng,xài luôn côsi+cân bằn hệ số.
bdt thứ 1 thì sai,thử với a=8,b=1/4,c=1/2



#239030 Chứng minh bđt sau

Đã gửi bởi tuan101293 on 01-09-2010 - 22:57 trong Bất đẳng thức - Cực trị

Đúng mà anh. LHS-RHS=2048.45...

đảo 1 tí là sai mà em
a=1/2,b=1/4,c=8 thì sai



#238995 Chứng minh bđt sau

Đã gửi bởi tuan101293 on 01-09-2010 - 20:36 trong Bất đẳng thức - Cực trị

Bài 1. Cho a,b,c là các số thực dương và abc=1.
Chứng minh :
$\dfrac{a}{a+{b}^{2}+{c}^{3}}+\dfrac{b}{b+{c}^{2}+{a}^{3}}+\dfrac{c}{c+{a}^{2}+{b}^{3}} \leq 1$
$ <=> \sum_{cyclic}[a(b+c^2+a^3)(c+a^2+b^3)] \leq (a+b^2+c^3)(b+c^2+a^3)(c+a^2+b^3) $
$ <=> \sum_{cyclic}a^{5}c^{3}+\sum_{cyclic}a^{5}b^{2}-\sum_{cyclic}a^{3}b-\sum_{cyclic}a^{4}c\geq 0 $
Không khó để chứng minh bởi AM-GM.

Ẹc,anh cũng rất quan tâm đến bài toán này,đã thử giải= cách quy đồng lâu rồi,được 1 nửa thì nửa còn lại sai,
@Nguyễn Thái Vũ,NOVAE,.....
NẾU MỌI NGƯờI ĐỌC KỸ THÌ ARQADY NÓI LÀ TÔI VẪN CHƯA GIẢI ĐƯỢC.VẬY MÀ MÌNH THẤY CÁC BẠN NÓI NHƯ LÀ HIỂU LỜI GIẢI CỦA ARQADY VÀ BÀO NÓ ĐÚNG.MÌNH CHỊU.............
@messi:Em giải được =amgm thì post lên cho anh được thưởng thức nhé :).thks em.



#219854 Tìm cực trị

Đã gửi bởi tuan101293 on 08-11-2009 - 17:50 trong Bất đẳng thức và cực trị

Với x như thế nào vậy? dương, âm hay nguyên dương?Nếu x nguyên dương thì ta có thể xét x=0 rồi với x nguyên dương ta có bđt$ \dfrac{a^x+b^x}{2} \geq (\dfrac{a+b}{2})^x$

với x>1 hoặc x<0 ta có $\dfrac{a^x+b^x}{2}\ge (\dfrac{a+b}{2})^x$
vơi 0<x<1 ta có $\dfrac{a^x+b^x}{2}\le (\dfrac{a+b}{2})^x$
với x=0,1 ta có đẳng thức



#237915 Inequality.

Đã gửi bởi tuan101293 on 22-08-2010 - 21:45 trong Bất đẳng thức - Cực trị

Cho a,b,c dương và $ ab+bc+ca=1 $. Chứng minh:
$ \dfrac{27}{4}(a+b)(b+c)(c+a) \geq (\sqrt{a+b}+\sqrt{b+c}+\sqrt{c+a})^2 $

$VT\le (\sqrt{6(a+b+c)})^2$
nên ta sẽ CM:$9\prod (a+b)\ge 8(\sum ab)(\sum a)$ (đúng)



#237967 Inequality.

Đã gửi bởi tuan101293 on 23-08-2010 - 11:16 trong Bất đẳng thức - Cực trị

$ am_a+bm_b+cm_c \leq \sqrt{bc}m_a+\sqrt{ca}m_b+\sqrt{ab}m_c $

Anh nghĩ 1 tiếng hoài chẳng ra,em post lời giải được ko?
@anh tú nói gì mà em chẳng hiểu.???:-?



#237671 Inequality.

Đã gửi bởi tuan101293 on 20-08-2010 - 07:30 trong Bất đẳng thức - Cực trị

áp dụng bunhia và svacs ta có
(Đặt $x= cos(\dfrac{A}{2}$,...,$a= sin(\dfrac{A}{2}$) ,chú ý $a+b+c\le \dfrac{3}{2}$
$VT=\sum \dfrac{x^4}{a^2}\ge \dfrac{1}{3}*(\sum\dfrac{x^2}{a})^2\ge \dfrac{1}{3}*(\dfrac{(x+y+z)^2}{a+b+c})^2\ge \dfrac{4(x+y+z)^4}{27}\ge 4xyz(x+y+z)$
ĐPCM



#245011 help me

Đã gửi bởi tuan101293 on 24-10-2010 - 09:24 trong Bất đẳng thức và cực trị

$\dfrac{a}{a^3+b^2+c}+\dfrac{b}{b^3+c^2+a}+\dfrac{c}{c^3+a^2+b}\le 1 $.

$\sum ac^2\le \sum\dfrac{a^2+c^2+c^2+1}{4}=\dfrac{3(a^2+b^2+c^2)+3}{4}(AG-GM)$

AM-GM kiểu gì thế?
phải xài cái
$\sum_{cyc}ac^2\le \sum a^3$ mới được chứ nhỉ?